Administración     

Olimpiadas de Matemáticas
Página de preparación y problemas

OME Local
OME Andaluza
OME Nacional
OIM
IMO
EGMO
USAMO
ASU
APMO
OMCC
Retos UJA
Selector
La base de datos contiene 2717 problemas y 972 soluciones.

XL Olimpiada Iberoamericana de Matemáticas — 2025

Sesión 1 —  25 de septiembre de 2025

Problema 2716
Una sucesión de números reales $a_1,a_2,\ldots$ se llama mapuche si $a_1\gt 0$ y además para todo $n\geq 2$ se tiene que \[a_1a_2\cdots a_n=a_1+a_2+\ldots+a_{n-1}.\] ¿Cuál es la máxima cantidad de enteros que puede tener una sucesión mapuche?

Nota. El producto tiene $n$ factores y la suma tiene $n-1$ sumandos.

Sin pistas
Sin soluciones
info
Si crees que el enunciado contiene un error o imprecisión o bien crees que la información sobre la procedencia del problema es incorrecta, puedes notificarlo usando los siguientes botones:
Informar de error en enunciado Informar de procedencia del problema
Problema 2717
Se tiene un tablero $n\times n$ dividido en $n^2$ casillas con $n\geq 3$. Inicialmente, se elige una casilla y se colocan en ella $n^2$ monedas. Un movimiento consiste en elegir una casilla que contenga al menos dos monedas y desplazar dos de ellas hacia dos casillas que sean simétricas con respecto a la casilla elegida y compartan al menos un vértice con ella. En la figura se muestran los cuatro tipos de movimientos posibles.

Si después de varios movimientos rsulta que en cada casilla del tablero hay exactamente una moneda, demostrar que la cantidad de movimientos realizados del tipo $3$ es igual a la cantidad de movimientos realizados del tipo $4$.

imagen
Sin pistas
Sin soluciones
info
Si crees que el enunciado contiene un error o imprecisión o bien crees que la información sobre la procedencia del problema es incorrecta, puedes notificarlo usando los siguientes botones:
Informar de error en enunciado Informar de procedencia del problema
Problema 2718
Sean $b$ y $n$ enteros positivos con $b\geq 2$. Se define $s_b(n)$ como la suma de las cifras de $n$ expresado en base $b$. ¿Existe algún entero $n\geq 2$ tal que \[s_2(n)\geq s_3(n)\geq\ldots\geq s_{2025}(n)?\]

Nota. Las cifras de $n$ expresado en base $b$ son los números enteros $a_0,a_1,\ldots,a_k$ tales que $n=a_0+a_1b+a_2b^2+\ldots+a_kb^k$ con $a_k\neq 0$ y $0\leq a_i\leq b-1$ para todo $i\in\{0,1,\ldots,k\}$.

Sin pistas
Sin soluciones
info
Si crees que el enunciado contiene un error o imprecisión o bien crees que la información sobre la procedencia del problema es incorrecta, puedes notificarlo usando los siguientes botones:
Informar de error en enunciado Informar de procedencia del problema

Sesión 2 —  26 de septiembre de 2025

Problema 2719
Encontrar todas las parejas de números primos $(p,q)$ con $p\gt q\gt 1$, tales que \[(p-q-1)^3+(p-q)^3+\ldots+(p-1)^3+p^3+\ldots+(p+q)^3+(p+q+1)^3=(3pq)^2.\]

Nota. El miembro de la izquierda de la igualdad tiene $2q+3$ sumandos, los cuales son cubos de números consecutivos.

Sin pistas
Sin soluciones
info
Si crees que el enunciado contiene un error o imprecisión o bien crees que la información sobre la procedencia del problema es incorrecta, puedes notificarlo usando los siguientes botones:
Informar de error en enunciado Informar de procedencia del problema
Problema 2720
El triángulo $ABC$ es acutángulo con $AB\lt AC$. Sean $\omega$ la circunferencia inscrita del triángulo $ABC$ y $\Gamma$ su circunferencia circunscrita. Sea $D$ el punto de tangencia de $\omega$ con el lado $BC$ y sea $L$ el punto de $\omega$ diametralmente opuesto a $D$. La recta $AL$ corta al lado $BC$ en el punto $E$. Sea $N$ el punto medio del arco $BC$ de $\Gamma$ que contiene a $A$. La recta $NL$ corta de nuevo a $\omega$ en el punto $K$. Demostrar que los puntos $A,N,E,K$ están en una misma circunferencia.
Sin pistas
Sin soluciones
info
Si crees que el enunciado contiene un error o imprecisión o bien crees que la información sobre la procedencia del problema es incorrecta, puedes notificarlo usando los siguientes botones:
Informar de error en enunciado Informar de procedencia del problema
Problema 2721
Un sultán tiene capturados a $23$ magos y les propone un juego para dejarlos libres. El sultán les dice que va a construir $11$ pozos, numerados del $1$ al $11$, y una torre. Dentro de cada poco pondrá a dos magos y pondrá al restante en la torre. A cada mago en los pozos le pondrá un sombrero de uno de los cuatro colores (conocidos por todos), y al de la torre le pondrá un sombrero de uno de $2025$ colores (distintos de los otros cuatro y conocidos por todos). Ningún mango sabrá el color de su sombrero. Una vez dentro del pozo, cada mago sabrá el número del pozi en el que está; además, verá únicamente el sombrero del mago de la torre y el del mago que el que compartirá pozo. El mago de la torre conocerá el número de cada pozo y podrá ver los sombreros de todos los demás magos.

En un determinado momento, el sultán dará la orden y, simultáneamente, cada mago dirá El color de mi sombrero es X, donde X es el color que quiera. Si al menos un mago dice una frase verdadera, todos los magos ganan y son libres; en otro caso, pierden. Antes de ser puestos en sus lugares y de recibir sus sombreros, los magos dispondrán de un tiempo para planear una estrategia, pero no podrán comunicarse después de esto. ¿Pueden asegurar la victoria sin importar lo que haga el sultán?

Sin pistas
Sin soluciones
info
Si crees que el enunciado contiene un error o imprecisión o bien crees que la información sobre la procedencia del problema es incorrecta, puedes notificarlo usando los siguientes botones:
Informar de error en enunciado Informar de procedencia del problema
José Miguel Manzano © 2010-2025. Esta página ha sido creada mediante software libre